Vertical force exerted by a spinning non-perpendicular gyroscope

AI Thread Summary
The wooden box with a 1 kg gyroscope inside maintains its weight on the scale at 1 kg, even when tilted at 45 degrees due to the gyroscopic effect. The gyroscope does not weigh less when spinning, as its mass remains constant and the forces acting on it do not change the total weight. The discussion highlights the importance of Newton's laws in understanding the acceleration of the center of mass and the forces involved. The gyroscope exhibits precession, where applied forces affect the gyroscope at a point 90 degrees along its rim from the point of application. This phenomenon occurs regardless of the rotational speed, demonstrating the unique dynamics of spinning objects.
aeroseek
Messages
49
Reaction score
0
A wooden box on a weighing scale weighs 1 kg. Inside the box is a gyroscope which is set spinning, and the box is now tilted at 45 degrees to the vertical and maintains this position due to the gyroscope inside.

What is the weight shown on the scale - is it 1 kg or less than 1 kg? Why is this?

Some contradictory answers here:

http://physics.stackexchange.com/questions/33866/does-a-toy-top-weigh-less-when-it-is-spinning
 
Physics news on Phys.org
Why would you think it would weigh less?
 
Apply Newtons laws. What is the acceleration of the center of mass? What force is required to make that acceleration?
 
Watch this video:

https://www.youtube.com/watch?v=GeyDf4ooPdo

https://www.youtube.com/watch?v=tLMpdBjA2SU
 
Last edited:
Great video A.T.! Thanks for posting. (Be sure to view the follow up explanatory videos. I remember the nonsense from Laithwaite some years back.)
 
Great video - I guess you guys can answer anything! Granted the gyroscope does not weigh less.

The question is then what effect does the gyroscope exhibit and why?

I used to simplify its effect as the following rule - push on the rim of a gyroscope and the force travels 90 degrees along the rim of the gyroscope in the direction of rotation before manifesting itself - any force takes effect not at the point of action but some place further along the rim - regardless of rotational speed? Why?
 
Thread 'Question about pressure of a liquid'
I am looking at pressure in liquids and I am testing my idea. The vertical tube is 100m, the contraption is filled with water. The vertical tube is very thin(maybe 1mm^2 cross section). The area of the base is ~100m^2. Will he top half be launched in the air if suddenly it cracked?- assuming its light enough. I want to test my idea that if I had a thin long ruber tube that I lifted up, then the pressure at "red lines" will be high and that the $force = pressure * area$ would be massive...
I feel it should be solvable we just need to find a perfect pattern, and there will be a general pattern since the forces acting are based on a single function, so..... you can't actually say it is unsolvable right? Cause imaging 3 bodies actually existed somwhere in this universe then nature isn't gonna wait till we predict it! And yea I have checked in many places that tiny changes cause large changes so it becomes chaos........ but still I just can't accept that it is impossible to solve...
Back
Top